Qustion about the sum of some specific irrational numbers












0












$begingroup$


I have been organizing my language, if there is anything not clear about the question, I am sorry.



The question should be:



$forall ninmathbb N^*, a_iinmathbb N$, $a_i$ is not a perfect square for $1leq ileq n$. Then $sum_{i=1}^n sqrt{a_i}$ is not a rational number.



(We assume every real number has a square root, and 0 is a perfect square.)



I saw the proof on the website as $n=2$, it is great, and very clever. And it can even prove $n=3$, this statement is true.



However, I think the key in the proof is to change all the square roots into rational numbers, so we get a contradiction. Yet when $n$ is bigger than $3$, square both sides will not be possible to reduce the number of square roots (at least I think). Thus it may not be effective from here.



I think the statement is true, because I think two positive irrational numbers sum up and get rational numbers will only be in form like $2-sqrt 2$ and $sqrt 2$,
and something like $2-sqrt 2$ cannot be $sqrt n$.



However, I cannot think of a way to prove it. This reminds me that when I want to prove something is a irrational number, all the way I have is to assume it is not, and get contradiction. I think this is not good, so if you have some other methods to prove something irrational, and willing to share with me, I will really be appreciated.










share|cite|improve this question











$endgroup$








  • 1




    $begingroup$
    Please learn to use MathJax to enter mathematical formulas. You can get help on this by clicking on the advanced help option in the window for entering and editing questions and following the link in the section entitled $LaTeX$.
    $endgroup$
    – Rob Arthan
    Jan 31 at 20:44










  • $begingroup$
    I've typeset the question for you, but please learn to do so yourself.
    $endgroup$
    – Henrik
    Jan 31 at 20:53










  • $begingroup$
    Thank you Henrik,I have been trying to learn to use LaTeX,but it did not go well.Next time I will try to type myself.
    $endgroup$
    – StAKmod
    Jan 31 at 20:55






  • 1




    $begingroup$
    This is definitely a true statement; but in this generality the most natural way to approach the proof would be via algebraic number theory, which is probably more advanced than you are hoping for. It would be nice if somebody could organize a more elementary proof.
    $endgroup$
    – Greg Martin
    Jan 31 at 21:26






  • 1




    $begingroup$
    Perhaps by strengthening the assertion one could proceed by induction—for example, proving that $sum_{i=1}^n sqrt{a_i}$ is not a root of any polynomial with integer coefficients of degree less than $2^n-1$ (being irrational is this statement for degree-$1$ polynomials). One would have to say something about the $a_i$ for this strengthening to be true, though—we couldn't allow $a_1=a_2=2$, or even $a_1=2,a_2=8$. But there is a way of measuring these dependencies and modifying the degree-bound $2^n-1$ accordingly.
    $endgroup$
    – Greg Martin
    Jan 31 at 21:27
















0












$begingroup$


I have been organizing my language, if there is anything not clear about the question, I am sorry.



The question should be:



$forall ninmathbb N^*, a_iinmathbb N$, $a_i$ is not a perfect square for $1leq ileq n$. Then $sum_{i=1}^n sqrt{a_i}$ is not a rational number.



(We assume every real number has a square root, and 0 is a perfect square.)



I saw the proof on the website as $n=2$, it is great, and very clever. And it can even prove $n=3$, this statement is true.



However, I think the key in the proof is to change all the square roots into rational numbers, so we get a contradiction. Yet when $n$ is bigger than $3$, square both sides will not be possible to reduce the number of square roots (at least I think). Thus it may not be effective from here.



I think the statement is true, because I think two positive irrational numbers sum up and get rational numbers will only be in form like $2-sqrt 2$ and $sqrt 2$,
and something like $2-sqrt 2$ cannot be $sqrt n$.



However, I cannot think of a way to prove it. This reminds me that when I want to prove something is a irrational number, all the way I have is to assume it is not, and get contradiction. I think this is not good, so if you have some other methods to prove something irrational, and willing to share with me, I will really be appreciated.










share|cite|improve this question











$endgroup$








  • 1




    $begingroup$
    Please learn to use MathJax to enter mathematical formulas. You can get help on this by clicking on the advanced help option in the window for entering and editing questions and following the link in the section entitled $LaTeX$.
    $endgroup$
    – Rob Arthan
    Jan 31 at 20:44










  • $begingroup$
    I've typeset the question for you, but please learn to do so yourself.
    $endgroup$
    – Henrik
    Jan 31 at 20:53










  • $begingroup$
    Thank you Henrik,I have been trying to learn to use LaTeX,but it did not go well.Next time I will try to type myself.
    $endgroup$
    – StAKmod
    Jan 31 at 20:55






  • 1




    $begingroup$
    This is definitely a true statement; but in this generality the most natural way to approach the proof would be via algebraic number theory, which is probably more advanced than you are hoping for. It would be nice if somebody could organize a more elementary proof.
    $endgroup$
    – Greg Martin
    Jan 31 at 21:26






  • 1




    $begingroup$
    Perhaps by strengthening the assertion one could proceed by induction—for example, proving that $sum_{i=1}^n sqrt{a_i}$ is not a root of any polynomial with integer coefficients of degree less than $2^n-1$ (being irrational is this statement for degree-$1$ polynomials). One would have to say something about the $a_i$ for this strengthening to be true, though—we couldn't allow $a_1=a_2=2$, or even $a_1=2,a_2=8$. But there is a way of measuring these dependencies and modifying the degree-bound $2^n-1$ accordingly.
    $endgroup$
    – Greg Martin
    Jan 31 at 21:27














0












0








0





$begingroup$


I have been organizing my language, if there is anything not clear about the question, I am sorry.



The question should be:



$forall ninmathbb N^*, a_iinmathbb N$, $a_i$ is not a perfect square for $1leq ileq n$. Then $sum_{i=1}^n sqrt{a_i}$ is not a rational number.



(We assume every real number has a square root, and 0 is a perfect square.)



I saw the proof on the website as $n=2$, it is great, and very clever. And it can even prove $n=3$, this statement is true.



However, I think the key in the proof is to change all the square roots into rational numbers, so we get a contradiction. Yet when $n$ is bigger than $3$, square both sides will not be possible to reduce the number of square roots (at least I think). Thus it may not be effective from here.



I think the statement is true, because I think two positive irrational numbers sum up and get rational numbers will only be in form like $2-sqrt 2$ and $sqrt 2$,
and something like $2-sqrt 2$ cannot be $sqrt n$.



However, I cannot think of a way to prove it. This reminds me that when I want to prove something is a irrational number, all the way I have is to assume it is not, and get contradiction. I think this is not good, so if you have some other methods to prove something irrational, and willing to share with me, I will really be appreciated.










share|cite|improve this question











$endgroup$




I have been organizing my language, if there is anything not clear about the question, I am sorry.



The question should be:



$forall ninmathbb N^*, a_iinmathbb N$, $a_i$ is not a perfect square for $1leq ileq n$. Then $sum_{i=1}^n sqrt{a_i}$ is not a rational number.



(We assume every real number has a square root, and 0 is a perfect square.)



I saw the proof on the website as $n=2$, it is great, and very clever. And it can even prove $n=3$, this statement is true.



However, I think the key in the proof is to change all the square roots into rational numbers, so we get a contradiction. Yet when $n$ is bigger than $3$, square both sides will not be possible to reduce the number of square roots (at least I think). Thus it may not be effective from here.



I think the statement is true, because I think two positive irrational numbers sum up and get rational numbers will only be in form like $2-sqrt 2$ and $sqrt 2$,
and something like $2-sqrt 2$ cannot be $sqrt n$.



However, I cannot think of a way to prove it. This reminds me that when I want to prove something is a irrational number, all the way I have is to assume it is not, and get contradiction. I think this is not good, so if you have some other methods to prove something irrational, and willing to share with me, I will really be appreciated.







number-theory discrete-mathematics






share|cite|improve this question















share|cite|improve this question













share|cite|improve this question




share|cite|improve this question








edited Jan 31 at 20:51









Henrik

6,04592030




6,04592030










asked Jan 31 at 20:35









StAKmodStAKmod

481111




481111








  • 1




    $begingroup$
    Please learn to use MathJax to enter mathematical formulas. You can get help on this by clicking on the advanced help option in the window for entering and editing questions and following the link in the section entitled $LaTeX$.
    $endgroup$
    – Rob Arthan
    Jan 31 at 20:44










  • $begingroup$
    I've typeset the question for you, but please learn to do so yourself.
    $endgroup$
    – Henrik
    Jan 31 at 20:53










  • $begingroup$
    Thank you Henrik,I have been trying to learn to use LaTeX,but it did not go well.Next time I will try to type myself.
    $endgroup$
    – StAKmod
    Jan 31 at 20:55






  • 1




    $begingroup$
    This is definitely a true statement; but in this generality the most natural way to approach the proof would be via algebraic number theory, which is probably more advanced than you are hoping for. It would be nice if somebody could organize a more elementary proof.
    $endgroup$
    – Greg Martin
    Jan 31 at 21:26






  • 1




    $begingroup$
    Perhaps by strengthening the assertion one could proceed by induction—for example, proving that $sum_{i=1}^n sqrt{a_i}$ is not a root of any polynomial with integer coefficients of degree less than $2^n-1$ (being irrational is this statement for degree-$1$ polynomials). One would have to say something about the $a_i$ for this strengthening to be true, though—we couldn't allow $a_1=a_2=2$, or even $a_1=2,a_2=8$. But there is a way of measuring these dependencies and modifying the degree-bound $2^n-1$ accordingly.
    $endgroup$
    – Greg Martin
    Jan 31 at 21:27














  • 1




    $begingroup$
    Please learn to use MathJax to enter mathematical formulas. You can get help on this by clicking on the advanced help option in the window for entering and editing questions and following the link in the section entitled $LaTeX$.
    $endgroup$
    – Rob Arthan
    Jan 31 at 20:44










  • $begingroup$
    I've typeset the question for you, but please learn to do so yourself.
    $endgroup$
    – Henrik
    Jan 31 at 20:53










  • $begingroup$
    Thank you Henrik,I have been trying to learn to use LaTeX,but it did not go well.Next time I will try to type myself.
    $endgroup$
    – StAKmod
    Jan 31 at 20:55






  • 1




    $begingroup$
    This is definitely a true statement; but in this generality the most natural way to approach the proof would be via algebraic number theory, which is probably more advanced than you are hoping for. It would be nice if somebody could organize a more elementary proof.
    $endgroup$
    – Greg Martin
    Jan 31 at 21:26






  • 1




    $begingroup$
    Perhaps by strengthening the assertion one could proceed by induction—for example, proving that $sum_{i=1}^n sqrt{a_i}$ is not a root of any polynomial with integer coefficients of degree less than $2^n-1$ (being irrational is this statement for degree-$1$ polynomials). One would have to say something about the $a_i$ for this strengthening to be true, though—we couldn't allow $a_1=a_2=2$, or even $a_1=2,a_2=8$. But there is a way of measuring these dependencies and modifying the degree-bound $2^n-1$ accordingly.
    $endgroup$
    – Greg Martin
    Jan 31 at 21:27








1




1




$begingroup$
Please learn to use MathJax to enter mathematical formulas. You can get help on this by clicking on the advanced help option in the window for entering and editing questions and following the link in the section entitled $LaTeX$.
$endgroup$
– Rob Arthan
Jan 31 at 20:44




$begingroup$
Please learn to use MathJax to enter mathematical formulas. You can get help on this by clicking on the advanced help option in the window for entering and editing questions and following the link in the section entitled $LaTeX$.
$endgroup$
– Rob Arthan
Jan 31 at 20:44












$begingroup$
I've typeset the question for you, but please learn to do so yourself.
$endgroup$
– Henrik
Jan 31 at 20:53




$begingroup$
I've typeset the question for you, but please learn to do so yourself.
$endgroup$
– Henrik
Jan 31 at 20:53












$begingroup$
Thank you Henrik,I have been trying to learn to use LaTeX,but it did not go well.Next time I will try to type myself.
$endgroup$
– StAKmod
Jan 31 at 20:55




$begingroup$
Thank you Henrik,I have been trying to learn to use LaTeX,but it did not go well.Next time I will try to type myself.
$endgroup$
– StAKmod
Jan 31 at 20:55




1




1




$begingroup$
This is definitely a true statement; but in this generality the most natural way to approach the proof would be via algebraic number theory, which is probably more advanced than you are hoping for. It would be nice if somebody could organize a more elementary proof.
$endgroup$
– Greg Martin
Jan 31 at 21:26




$begingroup$
This is definitely a true statement; but in this generality the most natural way to approach the proof would be via algebraic number theory, which is probably more advanced than you are hoping for. It would be nice if somebody could organize a more elementary proof.
$endgroup$
– Greg Martin
Jan 31 at 21:26




1




1




$begingroup$
Perhaps by strengthening the assertion one could proceed by induction—for example, proving that $sum_{i=1}^n sqrt{a_i}$ is not a root of any polynomial with integer coefficients of degree less than $2^n-1$ (being irrational is this statement for degree-$1$ polynomials). One would have to say something about the $a_i$ for this strengthening to be true, though—we couldn't allow $a_1=a_2=2$, or even $a_1=2,a_2=8$. But there is a way of measuring these dependencies and modifying the degree-bound $2^n-1$ accordingly.
$endgroup$
– Greg Martin
Jan 31 at 21:27




$begingroup$
Perhaps by strengthening the assertion one could proceed by induction—for example, proving that $sum_{i=1}^n sqrt{a_i}$ is not a root of any polynomial with integer coefficients of degree less than $2^n-1$ (being irrational is this statement for degree-$1$ polynomials). One would have to say something about the $a_i$ for this strengthening to be true, though—we couldn't allow $a_1=a_2=2$, or even $a_1=2,a_2=8$. But there is a way of measuring these dependencies and modifying the degree-bound $2^n-1$ accordingly.
$endgroup$
– Greg Martin
Jan 31 at 21:27










0






active

oldest

votes












Your Answer





StackExchange.ifUsing("editor", function () {
return StackExchange.using("mathjaxEditing", function () {
StackExchange.MarkdownEditor.creationCallbacks.add(function (editor, postfix) {
StackExchange.mathjaxEditing.prepareWmdForMathJax(editor, postfix, [["$", "$"], ["\\(","\\)"]]);
});
});
}, "mathjax-editing");

StackExchange.ready(function() {
var channelOptions = {
tags: "".split(" "),
id: "69"
};
initTagRenderer("".split(" "), "".split(" "), channelOptions);

StackExchange.using("externalEditor", function() {
// Have to fire editor after snippets, if snippets enabled
if (StackExchange.settings.snippets.snippetsEnabled) {
StackExchange.using("snippets", function() {
createEditor();
});
}
else {
createEditor();
}
});

function createEditor() {
StackExchange.prepareEditor({
heartbeatType: 'answer',
autoActivateHeartbeat: false,
convertImagesToLinks: true,
noModals: true,
showLowRepImageUploadWarning: true,
reputationToPostImages: 10,
bindNavPrevention: true,
postfix: "",
imageUploader: {
brandingHtml: "Powered by u003ca class="icon-imgur-white" href="https://imgur.com/"u003eu003c/au003e",
contentPolicyHtml: "User contributions licensed under u003ca href="https://creativecommons.org/licenses/by-sa/3.0/"u003ecc by-sa 3.0 with attribution requiredu003c/au003e u003ca href="https://stackoverflow.com/legal/content-policy"u003e(content policy)u003c/au003e",
allowUrls: true
},
noCode: true, onDemand: true,
discardSelector: ".discard-answer"
,immediatelyShowMarkdownHelp:true
});


}
});














draft saved

draft discarded


















StackExchange.ready(
function () {
StackExchange.openid.initPostLogin('.new-post-login', 'https%3a%2f%2fmath.stackexchange.com%2fquestions%2f3095420%2fqustion-about-the-sum-of-some-specific-irrational-numbers%23new-answer', 'question_page');
}
);

Post as a guest















Required, but never shown

























0






active

oldest

votes








0






active

oldest

votes









active

oldest

votes






active

oldest

votes
















draft saved

draft discarded




















































Thanks for contributing an answer to Mathematics Stack Exchange!


  • Please be sure to answer the question. Provide details and share your research!

But avoid



  • Asking for help, clarification, or responding to other answers.

  • Making statements based on opinion; back them up with references or personal experience.


Use MathJax to format equations. MathJax reference.


To learn more, see our tips on writing great answers.




draft saved


draft discarded














StackExchange.ready(
function () {
StackExchange.openid.initPostLogin('.new-post-login', 'https%3a%2f%2fmath.stackexchange.com%2fquestions%2f3095420%2fqustion-about-the-sum-of-some-specific-irrational-numbers%23new-answer', 'question_page');
}
);

Post as a guest















Required, but never shown





















































Required, but never shown














Required, but never shown












Required, but never shown







Required, but never shown

































Required, but never shown














Required, but never shown












Required, but never shown







Required, but never shown







Popular posts from this blog

Can a sorcerer learn a 5th-level spell early by creating spell slots using the Font of Magic feature?

Does disintegrating a polymorphed enemy still kill it after the 2018 errata?

A Topological Invariant for $pi_3(U(n))$